Live Discussion!
2025 MATHCOUNTS State Discussion is going on now!

Stay ahead of learning milestones! Enroll in a class over the summer!

G
Topic
First Poster
Last Poster
k a April Highlights and 2025 AoPS Online Class Information
jlacosta   0
Apr 2, 2025
Spring is in full swing and summer is right around the corner, what are your plans? At AoPS Online our schedule has new classes starting now through July, so be sure to keep your skills sharp and be prepared for the Fall school year! Check out the schedule of upcoming classes below.

WOOT early bird pricing is in effect, don’t miss out! If you took MathWOOT Level 2 last year, no worries, it is all new problems this year! Our Worldwide Online Olympiad Training program is for high school level competitors. AoPS designed these courses to help our top students get the deep focus they need to succeed in their specific competition goals. Check out the details at this link for all our WOOT programs in math, computer science, chemistry, and physics.

Looking for summer camps in math and language arts? Be sure to check out the video-based summer camps offered at the Virtual Campus that are 2- to 4-weeks in duration. There are middle and high school competition math camps as well as Math Beasts camps that review key topics coupled with fun explorations covering areas such as graph theory (Math Beasts Camp 6), cryptography (Math Beasts Camp 7-8), and topology (Math Beasts Camp 8-9)!

Be sure to mark your calendars for the following events:
[list][*]April 3rd (Webinar), 4pm PT/7:00pm ET, Learning with AoPS: Perspectives from a Parent, Math Camp Instructor, and University Professor
[*]April 8th (Math Jam), 4:30pm PT/7:30pm ET, 2025 MATHCOUNTS State Discussion
April 9th (Webinar), 4:00pm PT/7:00pm ET, Learn about Video-based Summer Camps at the Virtual Campus
[*]April 10th (Math Jam), 4:30pm PT/7:30pm ET, 2025 MathILy and MathILy-Er Math Jam: Multibackwards Numbers
[*]April 22nd (Webinar), 4:00pm PT/7:00pm ET, Competitive Programming at AoPS (USACO).[/list]
Our full course list for upcoming classes is below:
All classes run 7:30pm-8:45pm ET/4:30pm - 5:45pm PT unless otherwise noted.

Introductory: Grades 5-10

Prealgebra 1 Self-Paced

Prealgebra 1
Sunday, Apr 13 - Aug 10
Tuesday, May 13 - Aug 26
Thursday, May 29 - Sep 11
Sunday, Jun 15 - Oct 12
Monday, Jun 30 - Oct 20
Wednesday, Jul 16 - Oct 29

Prealgebra 2 Self-Paced

Prealgebra 2
Sunday, Apr 13 - Aug 10
Wednesday, May 7 - Aug 20
Monday, Jun 2 - Sep 22
Sunday, Jun 29 - Oct 26
Friday, Jul 25 - Nov 21

Introduction to Algebra A Self-Paced

Introduction to Algebra A
Monday, Apr 7 - Jul 28
Sunday, May 11 - Sep 14 (1:00 - 2:30 pm ET/10:00 - 11:30 am PT)
Wednesday, May 14 - Aug 27
Friday, May 30 - Sep 26
Monday, Jun 2 - Sep 22
Sunday, Jun 15 - Oct 12
Thursday, Jun 26 - Oct 9
Tuesday, Jul 15 - Oct 28

Introduction to Counting & Probability Self-Paced

Introduction to Counting & Probability
Wednesday, Apr 16 - Jul 2
Thursday, May 15 - Jul 31
Sunday, Jun 1 - Aug 24
Thursday, Jun 12 - Aug 28
Wednesday, Jul 9 - Sep 24
Sunday, Jul 27 - Oct 19

Introduction to Number Theory
Thursday, Apr 17 - Jul 3
Friday, May 9 - Aug 1
Wednesday, May 21 - Aug 6
Monday, Jun 9 - Aug 25
Sunday, Jun 15 - Sep 14
Tuesday, Jul 15 - Sep 30

Introduction to Algebra B Self-Paced

Introduction to Algebra B
Wednesday, Apr 16 - Jul 30
Tuesday, May 6 - Aug 19
Wednesday, Jun 4 - Sep 17
Sunday, Jun 22 - Oct 19
Friday, Jul 18 - Nov 14

Introduction to Geometry
Wednesday, Apr 23 - Oct 1
Sunday, May 11 - Nov 9
Tuesday, May 20 - Oct 28
Monday, Jun 16 - Dec 8
Friday, Jun 20 - Jan 9
Sunday, Jun 29 - Jan 11
Monday, Jul 14 - Jan 19

Intermediate: Grades 8-12

Intermediate Algebra
Monday, Apr 21 - Oct 13
Sunday, Jun 1 - Nov 23
Tuesday, Jun 10 - Nov 18
Wednesday, Jun 25 - Dec 10
Sunday, Jul 13 - Jan 18
Thursday, Jul 24 - Jan 22

Intermediate Counting & Probability
Wednesday, May 21 - Sep 17
Sunday, Jun 22 - Nov 2

Intermediate Number Theory
Friday, Apr 11 - Jun 27
Sunday, Jun 1 - Aug 24
Wednesday, Jun 18 - Sep 3

Precalculus
Wednesday, Apr 9 - Sep 3
Friday, May 16 - Oct 24
Sunday, Jun 1 - Nov 9
Monday, Jun 30 - Dec 8

Advanced: Grades 9-12

Olympiad Geometry
Tuesday, Jun 10 - Aug 26

Calculus
Tuesday, May 27 - Nov 11
Wednesday, Jun 25 - Dec 17

Group Theory
Thursday, Jun 12 - Sep 11

Contest Preparation: Grades 6-12

MATHCOUNTS/AMC 8 Basics
Wednesday, Apr 16 - Jul 2
Friday, May 23 - Aug 15
Monday, Jun 2 - Aug 18
Thursday, Jun 12 - Aug 28
Sunday, Jun 22 - Sep 21
Tues & Thurs, Jul 8 - Aug 14 (meets twice a week!)

MATHCOUNTS/AMC 8 Advanced
Friday, Apr 11 - Jun 27
Sunday, May 11 - Aug 10
Tuesday, May 27 - Aug 12
Wednesday, Jun 11 - Aug 27
Sunday, Jun 22 - Sep 21
Tues & Thurs, Jul 8 - Aug 14 (meets twice a week!)

AMC 10 Problem Series
Friday, May 9 - Aug 1
Sunday, Jun 1 - Aug 24
Thursday, Jun 12 - Aug 28
Tuesday, Jun 17 - Sep 2
Sunday, Jun 22 - Sep 21 (1:00 - 2:30 pm ET/10:00 - 11:30 am PT)
Monday, Jun 23 - Sep 15
Tues & Thurs, Jul 8 - Aug 14 (meets twice a week!)

AMC 10 Final Fives
Sunday, May 11 - Jun 8
Tuesday, May 27 - Jun 17
Monday, Jun 30 - Jul 21

AMC 12 Problem Series
Tuesday, May 27 - Aug 12
Thursday, Jun 12 - Aug 28
Sunday, Jun 22 - Sep 21
Wednesday, Aug 6 - Oct 22

AMC 12 Final Fives
Sunday, May 18 - Jun 15

F=ma Problem Series
Wednesday, Jun 11 - Aug 27

WOOT Programs
Visit the pages linked for full schedule details for each of these programs!


MathWOOT Level 1
MathWOOT Level 2
ChemWOOT
CodeWOOT
PhysicsWOOT

Programming

Introduction to Programming with Python
Thursday, May 22 - Aug 7
Sunday, Jun 15 - Sep 14 (1:00 - 2:30 pm ET/10:00 - 11:30 am PT)
Tuesday, Jun 17 - Sep 2
Monday, Jun 30 - Sep 22

Intermediate Programming with Python
Sunday, Jun 1 - Aug 24
Monday, Jun 30 - Sep 22

USACO Bronze Problem Series
Tuesday, May 13 - Jul 29
Sunday, Jun 22 - Sep 1

Physics

Introduction to Physics
Wednesday, May 21 - Aug 6
Sunday, Jun 15 - Sep 14
Monday, Jun 23 - Sep 15

Physics 1: Mechanics
Thursday, May 22 - Oct 30
Monday, Jun 23 - Dec 15

Relativity
Sat & Sun, Apr 26 - Apr 27 (4:00 - 7:00 pm ET/1:00 - 4:00pm PT)
Mon, Tue, Wed & Thurs, Jun 23 - Jun 26 (meets every day of the week!)
0 replies
jlacosta
Apr 2, 2025
0 replies
Cyclic Points
IstekOlympiadTeam   38
N 18 minutes ago by eg4334
Source: EGMO 2017 Day1 P1
Let $ABCD$ be a convex quadrilateral with $\angle DAB=\angle BCD=90^{\circ}$ and $\angle ABC> \angle CDA$. Let $Q$ and $R$ be points on segments $BC$ and $CD$, respectively, such that line $QR$ intersects lines $AB$ and $AD$ at points $P$ and $S$, respectively. It is given that $PQ=RS$.Let the midpoint of $BD$ be $M$ and the midpoint of $QR$ be $N$.Prove that the points $M,N,A$ and $C$ lie on a circle.
38 replies
IstekOlympiadTeam
Apr 8, 2017
eg4334
18 minutes ago
2025 Caucasus MO Seniors P3
BR1F1SZ   1
N 31 minutes ago by iliya8788
Source: Caucasus MO
A circle is drawn on the board, and $2n$ points are marked on it, dividing it into $2n$ equal arcs. Petya and Vasya are playing the following game. Petya chooses a positive integer $d \leqslant n$ and announces this number to Vasya. To win the game, Vasya needs to color all marked points using $n$ colors, such that each color is assigned to exactly two points, and for each pair of same-colored points, one of the arcs between them contains exactly $(d - 1)$ marked points. Find all $n$ for which Petya will be able to prevent Vasya from winning.
1 reply
BR1F1SZ
Mar 26, 2025
iliya8788
31 minutes ago
RGB chessboard
BR1F1SZ   0
an hour ago
Source: 2025 Argentina TST P3
A $100 \times 100$ board has some of its cells coloured red, blue, or green. Each cell is coloured with at most one colour, and some cells may remain uncoloured. Additionally, there is at least one cell of each colour. Two coloured cells are said to be friends if they have different colours and lie in the same row or in the same column. The following conditions are satisfied:
[list=i]
[*]Each coloured cell has exactly three friends.
[*]All three friends of any given coloured cell lie in the same row or in the same column.
[/list]
Determine the maximum number of cells that can be coloured on the board.
0 replies
BR1F1SZ
an hour ago
0 replies
[ELMO2] The Multiplication Table
v_Enhance   26
N an hour ago by de-Kirschbaum
Source: ELMO 2015, Problem 2 (Shortlist N1)
Let $m$, $n$, and $x$ be positive integers. Prove that \[ \sum_{i = 1}^n \min\left(\left\lfloor \frac{x}{i} \right\rfloor, m \right) = \sum_{i = 1}^m \min\left(\left\lfloor \frac{x}{i} \right\rfloor, n \right). \]
Proposed by Yang Liu
26 replies
v_Enhance
Jun 27, 2015
de-Kirschbaum
an hour ago
No more topics!
Prove that XY // BC
cauchyguy   32
N Jul 10, 2024 by cursed_tangent1434
Source: 2004 USA TST
Let $ABC$ be a triangle. Choose a point $D$ in its interior. Let $\omega_1$ be a circle passing through $B$ and $D$ and $\omega_2$ be a circle passing through $C$ and $D$ so that the other point of intersection of the two circles lies on $AD$. Let $\omega_1$ and $\omega_2$ intersect side $BC$ at $E$ and $F$, respectively. Denote by $X$ the intersection of $DF$, $AB$ and $Y$ the intersection of $DE, AC$. Show that $XY \parallel BC$.
32 replies
cauchyguy
Mar 18, 2006
cursed_tangent1434
Jul 10, 2024
Prove that XY // BC
G H J
Source: 2004 USA TST
The post below has been deleted. Click to close.
This post has been deleted. Click here to see post.
cauchyguy
61 posts
#1 • 6 Y
Y by nguyendangkhoa17112003, Adventure10, son7, Mango247, crazyeyemoody907, and 1 other user
Let $ABC$ be a triangle. Choose a point $D$ in its interior. Let $\omega_1$ be a circle passing through $B$ and $D$ and $\omega_2$ be a circle passing through $C$ and $D$ so that the other point of intersection of the two circles lies on $AD$. Let $\omega_1$ and $\omega_2$ intersect side $BC$ at $E$ and $F$, respectively. Denote by $X$ the intersection of $DF$, $AB$ and $Y$ the intersection of $DE, AC$. Show that $XY \parallel BC$.
Z K Y
The post below has been deleted. Click to close.
This post has been deleted. Click here to see post.
Sailor
256 posts
#2 • 1 Y
Y by Adventure10
Let $AB\cap{\omega_1}=\{X'\}$ and $AC\cap{\omega_2}=\{Y'\}$.
Then $\angle{X'DY}=\angle{ABC}=\angle{YY'X'}$ thus $X'DY'Y$ is cyclic. Analogously, $XX'DY'$ is cyclic too.
Consequently $X,Y,Y',D,X'$ all lie on a circle. From here we obatin that $XY$ and $X'Y\"$ are antiparallel.
Therefore $XY||BC$.
Attachments:
Z K Y
The post below has been deleted. Click to close.
This post has been deleted. Click here to see post.
epitomy01
240 posts
#3 • 2 Y
Y by Adventure10, Mango247
We can also use Menalaus and some quick computations:
Let $ R$ be the second point of intersection of the two circles, which lies on $ AD$.
Applying Menelaus to collinear points $ X, D, F$ in triangle $ ABS$ gives us:
$ \frac {AX}{XB} = \frac {AD}{SD} * \frac {SF}{BF} ... (1)$.
Applying Menelaus to collinear points $ E, D, Y$ in triangle $ ACS$ gives us:
$ \frac {AY}{CY} = \frac {AD}{SD} * \frac {SE}{CE} ... (2)$.
To prove $ XY$ is parallel to $ BC$, we must prove that $ \frac {AX}{XB} = \frac {AY}{CY}$ - using $ (1), (2)$ that is equivalent to proving:
$ \frac {SF}{BF} = \frac {SE}{CE}$, which is true since:
$ SF*CE - SE*BF = SF(SC+SE) - SE(BS+SF) = SF*SC - SE*SB = SR*RD - SR*RD = 0$.
Here we have used Power of a point to see that $ SF*SC = SR*RD = SE*SB$.
Z K Y
The post below has been deleted. Click to close.
This post has been deleted. Click here to see post.
v_Enhance
6871 posts
#4 • 6 Y
Y by anser, myh2910, Minkowsi47, Adventure10, Mango247, and 1 other user
We use barycentrics on $\triangle DEF$. Let $D = (1,0,0)$, $E = (0,1,0)$ and $F = (0,0,1)$ and define $a=EF$, $b=FD$ and $c=DE$. Set $D_1 = (u:m:n)$ and $A = (v:m:n)$, where $D_1$ is the second intersection of $\omega_1$ and $\omega_2$. Let $L = \tfrac{-a^2mn-b^2nu-c^2um}{u+m+n}$. (Guess what? $u$ never appears again, because the degree of freedom described by $u$ is encoded in $L$.)

Then, we find that \[ (DED_1) : -a^2yz-b^2zx-c^2xy + \left( x+y+z \right)\left( \frac{L}{n} z  \right) = 0. \]
Now we obtain $B$ by setting $x=0$ and dividing through by $z \neq 0$ (so that $B \neq E$) to give $(y+z) \frac{L}{n} = a^2y$; thus, we find that \[ B = (0 : L : na^2 - L). \] Then computing $X = DF \cap AB$, which is nice since $DF : y=0$, we find that \[ 
	X = \left( -\det \left[ \begin{array}{cc} v & m \\ 0 &n \end{array} \right]
		: 0
		: \det \left[ \begin{array}{cc} m & n \\ L & na^2 - L \end{array} \right] \right)
\] That is, \[ X = \left( -vL: 0 : mna^2-mL-nL \right). \] Similarly, $Y = \left( -vL : mna^2-mL-nL : 0 \right)$. So $XY \parallel EF$ and we're done.
Z K Y
The post below has been deleted. Click to close.
This post has been deleted. Click here to see post.
subham1729
1479 posts
#5 • 2 Y
Y by Adventure10, Mango247
Suppose $B=(-1,0),F=(a,0),E=(b,0),C=(1,0)$ and eq of circles are $x^2+y^2+2g_1x+2h_1y=1-2g_1$ and $x^2+y^2+2g_2x+2h_2y=1+2g_2$. Now certainly we've $g_1=-\frac {1+a}{2},g_2=\frac {1-b}{2}$. Now equation of line $AD$ be $2x(g_1-g_2)+2y(h_1-h_2)+c_1-c_2=0$ so intersection point of $AD,BC$ is $S=(-\frac {c_1-c_2}{h_1-h_2},0)$. Now so finally we get $\frac {BF}{SF}=\frac {SE}{ES}$, and now just by menalaus we're done.
Z K Y
The post below has been deleted. Click to close.
This post has been deleted. Click here to see post.
vanu1996
607 posts
#6 • 1 Y
Y by Adventure10
$\omega_1$ and $\omega_2$ intersects each other at $D,E$,and $AB,AC$ at $K$ and $L$,now $AE.AD=AL.AC$ and $AE.AD=AK.AB$,hence we get $KBCL$ is cyclic.so $\angle FDL=\angle BKL$ and $\angle EDK=\angle CLK$ ,so $KLYDX$ is cyclic. so $\angle DLY=\angle DXY$,also $\angle DLY=\angle EFD$,hence done.
Z K Y
The post below has been deleted. Click to close.
This post has been deleted. Click here to see post.
JuanOrtiz
366 posts
#7 • 2 Y
Y by Adventure10, Mango247
Let $w_1 \cap AB =G$, $w_2 \cap AC=H$. Let $Y'$ be the point on $AC$ such that $XY' || BC$. Notice $\angle Y'HD = \angle DFB = \angle DXY'$ and so $XDY'H$ is cyclic. Notice also $\frac{AX}{AY'} = \frac{AB}{AC} = \frac{AH}{AG}$ since $GHBC$ is cyclic. Therefore $GHDXY'$ is cyclic. So $\triangle GHD$'s circumcircle passes through $X$, and through $Y$ analogously. Then $Y'=Y$ so we're done.
Z K Y
The post below has been deleted. Click to close.
This post has been deleted. Click here to see post.
jayme
9775 posts
#8 • 1 Y
Y by Adventure10
Dear Mathlinkers,
another approach withe the Reim's theorem...
Sincerely
Jean-Louis
Z K Y
The post below has been deleted. Click to close.
This post has been deleted. Click here to see post.
jlammy
1099 posts
#9 • 2 Y
Y by Adventure10, Mango247
jayme,

Let $AB\cap{\omega_1}=\{B,X'\}, AC\cap{\omega_2}=\{C,Y'\}$. Then $\angle{X'DY}=\angle{ABC}=\angle{YY'X'}$, so $X'DY'Y$ is cyclic. Analogously, $XX'DY'$ is cyclic too.By Reim's theorem on $\{(XYD), \omega_1 \}$, $XY \parallel BE$, as required.

Alternatively, we may apply Reim's theorem on $\{(XYD), \omega_ 2\}$, so $XY \parallel FC$, as required. This solution is, of course, only a breath away from Sailor's solution.
Z K Y
The post below has been deleted. Click to close.
This post has been deleted. Click here to see post.
Dukejukem
695 posts
#10 • 2 Y
Y by Adventure10, Mango247
Here is another solution: We work in the projective plane. Denote by $D' \in AD$ the second intersection of $\omega_1$ and $\omega_2.$ Let $R = DX \cap D'B$ and $S = DY \cap D'C.$ Then using directed angles, we have that
\begin{align*} \measuredangle RD'S &= \measuredangle BD'C = \measuredangle BD'D + \measuredangle DD'C \\
&= \measuredangle BED + \measuredangle DFC = \measuredangle FED + \measuredangle DFC \\
&= -\measuredangle EDF = -\measuredangle SDR = \measuredangle RDS.
\end{align*} Therefore, we have that $\measuredangle RD'S = \measuredangle RDS$, implying that $R, S, D, D'$ are concyclic. It follows that \[\measuredangle D'RS = \measuredangle D'DS = \measuredangle D'DE = \measuredangle D'BE = \measuredangle D'BC.\] Then, since $\measuredangle D'RS = \measuredangle D'BC$, we conclude that $RS \parallel BC.$

Now, consider the lines $DD', XB, YC.$ Since these three lines all concur at one point (that is, $A$), we deduce that $\triangle DXY$ and $\triangle D'BC$ are in perspective. By Desargue's Theorem, it follows that $R = DX \cap D'B, \; S = DY \cap D'C, \; XY \cap BC$ are collinear. Therefore, $XY \cap BC$ lies on $RS$, which is a line parallel to $BC.$ It follows that $XY \parallel BC$, as desired. $\square$
Z K Y
The post below has been deleted. Click to close.
This post has been deleted. Click here to see post.
EulerMacaroni
851 posts
#11 • 2 Y
Y by Adventure10, Mango247
If I'm not mistaken, this problem follows directly from angle chasing.

Suppose $\omega_1 \cap AB=M$ and $\omega_2 \cap AC=P$; we claim $BPMC$ is cyclic. This follows quickly from Power of a Point; let the second intersection of the two circles be $D'$, then $AD\cdot AD'=AM\cdot AB= AP \cdot AC$.

Now, we will prove that $XMDPY$ is cyclic. It suffices to show that $XMDY$ is cyclic, since the total conclusion follows from symmetry. Remark that $\angle XDM=180^{\circ}-\angle MDE=\angle B$ and $\angle XPM=180^{\circ}-\angle MPC=\angle B$ by the previously established cyclicity.

To prove the parallelism, we check that $\angle YXE=\angle FED$. Angle chasing yields
$$\angle YXD=\angle YMD=180^{\circ}-\angle BMD=\angle BED=\angle FED$$ as desired$.\;\blacksquare$
Z K Y
The post below has been deleted. Click to close.
This post has been deleted. Click here to see post.
K6160
276 posts
#12 • 2 Y
Y by Adventure10, Mango247
Let the second intersections of $\omega_1$ and $\omega_2$ with sides $AB$ and $AC$ be $X'$ and $Y'$. We claim that $XX'DYY'$ is cyclic. This is true since $X'Y'D=180-\angle B-\angle DY'C=180-\angle B-\angle DFE=\angle X'XD$. However, we know that $X'Y'CB$ is cyclic. So $XY$ is antiparallel to $X'Y'$, which is antiparallel to $BC$. It follows that $XY\parallel BC$. Other configurations can be handled similarly.$\Box$
Z K Y
The post below has been deleted. Click to close.
This post has been deleted. Click here to see post.
Ankoganit
3070 posts
#13 • 2 Y
Y by Adventure10, Mango247
We shall use the configuration depicted below. Other cases are "similarly" handled. [asy]import graph; size(10cm); real lsf=0.5; pathpen=linewidth(0.7); pointpen=black; pen fp=fontsize(10); pointfontpen=fp; real xmin=-3.715082517158455,xmax=8.57101545322826,ymin=-0.8000376867959583,ymax=6.082084462204611; 
pen ubqqys=rgb(0.29411764705882354,0.,0.5098039215686274), uuuuuu=rgb(0.26666666666666666,0.26666666666666666,0.26666666666666666), yqqqqq=rgb(0.5019607843137255,0.,0.), qqffff=rgb(0.,1.,1.); 
pair A=(0.4149689204417927,5.0446348930406835), B=(-0.8700481876639894,0.45559198934964207), C=(4.982241647714512,0.4482712566770508), D=(1.0490909090909089,2.889090909090912), X_1=(1.8477622637606381,0.17420124349384086), F=(1.537023192408008,0.4525809412653818), X=(0.5330076578212042,5.4661738956848795), Y=(-0.004570323506018968,5.466846361472242), T=(1.6558161096067847,3.7958842185943857); 
D(A--B--C--cycle,blue); 
D(A--B,blue); D(B--C,blue); D(C--A,blue); D(CR((0.5886467082123572,1.2787144125300975),1.6749093476359163),gray); D(CR((3.2616521993693435,2.0650642018842325),2.3610267845639625),gray); D(Y--X,blue); D(CR((0.26247397537495465,4.071780606014076),1.420394705571974),yellow); D(T--D,yqqqqq); D((-0.2304653826081835,2.739665029583861)--T); D(A--X_1,linetype("2 2")+qqffff); D(Y--A); D(X--A); D(F--X,yqqqqq); D((2.0452777254599024,0.4519451566971582)--Y,yqqqqq); D((-0.2304653826081835,2.739665029583861)--D,yqqqqq); 
D(A,linewidth(3.pt)+blue); MP("A",(0.6,5.017183344460874),NE*lsf,fp+blue); D(B,linewidth(3.pt)+blue); MP("B",(-0.8064720015300348,0.5509562685505738),NE*lsf,fp+blue); D(C,linewidth(3.pt)+blue); MP("C",(5.0425371227937825,0.5509562685505738),NE*lsf,fp+blue); D(D,linewidth(3.pt)+red); MP("D",(1.2120719082230216,2.8238049228394453),NE*lsf,fp+red); D((2.0452777254599024,0.4519451566971582),linewidth(3.pt)+ubqqys); MP("E",(2.149820653698851,0.4873800824166193),NE*lsf,fp+ubqqys); MP("\omega_1",(-1.3468695836686484,1.7589038050957084),NE*lsf,fp+gray); D(X_1,linewidth(3.pt)+uuuuuu); MP("\omega_2",(3.3418741437104984,3.6820834356478307),NE*lsf,fp+gray); D(F,linewidth(3.pt)+ubqqys); MP("F",(1.545846885426283,0.4873800824166193),NE*lsf,fp+ubqqys); D(X,linewidth(3.pt)+ubqqys); MP("X",(0.5922040934169649,5.557580926599487),NE*lsf,fp+ubqqys); D(Y,linewidth(3.pt)+ubqqys); MP("Y",(0.05180651127835134,5.557580926599487),NE*lsf,fp+ubqqys); D((-0.2304653826081835,2.739665029583861),linewidth(3.pt)+uuuuuu); MP("S",(-0.45680297779328494,2.8238049228394453),NE*lsf,fp+uuuuuu); D(T,linewidth(3.pt)+uuuuuu); MP("T",(1.720681397294658,3.8887060405831826),NE*lsf,fp+uuuuuu); 
clip((xmin,ymin)--(xmin,ymax)--(xmax,ymax)--(xmax,ymin)--cycle); [/asy]
Let $S$ and $T$ be the second intersections of $\omega _1$ with $AB$ and $\omega _2$ with $AC$ respectively. Note that $A$ lies on the radical axis of the circles $\omega _1$ and $\omega _2$, so $A$ has same power wrt these circles. Thus we have $AS\cdot AB=AT\cdot AC$, so that $STCB$ is cyclic. Now we have the following equality of angles:\begin{align*} \angle SXD=&\angle BXF\\
=&\angle XFC-\angle XBF\\
=&\angle DFC-\angle SBC\\
=&\angle DTA-\angle STA\qquad\text{[since quadrilaterals DFCT and SBCT are cyclic]}\\
=&\angle STD\end{align*}Thus points $S,X,D,T$ are concyclic. Similarly it may be shown that $S,Y,D,T$ are concyclic; thus it follows that $S,T,X,Y$ are concyclic (all lie on the circumcircle of $\triangle SDT$). Then by power of point we have $$AY\cdot AT=AX\cdot AS\implies \frac{AX}{AY}=\frac{AT}{AS}\quad (1).$$From cyclic quadrilateral $SBCT$, we also have $$AS\cdot AB=AT\cdot AC\implies \frac{AB}{AC}=\frac{AT}{AS}\quad (2).$$From $(1)$ and $(2)$, we obtain that $\frac{AX}{AY}=\frac{AB}{AC}$. By the converse of Thales' Theorem, this implies $XY||BC$, as desired. $\blacksquare$
This post has been edited 2 times. Last edited by Ankoganit, Mar 26, 2016, 5:00 AM
Z K Y
The post below has been deleted. Click to close.
This post has been deleted. Click here to see post.
K.N
532 posts
#14 • 1 Y
Y by Adventure10
JuanOrtiz wrote:
Let $w_1 \cap AB =G$, $w_2 \cap AC=H$. Let $Y'$ be the point on $AC$ such that $XY' || BC$. Notice $\angle Y'HD = \angle DFB = \angle DXY'$ and so $XDY'H$ is cyclic. Notice also $\frac{AX}{AY'} = \frac{AB}{AC} = \frac{AH}{AG}$ since $GHBC$ is cyclic. Therefore $GHDXY'$ is cyclic. So $\triangle GHD$'s circumcircle passes through $X$, and through $Y$ analogously. Then $Y'=Y$ so we're done.

That's exactly my solution too
Nice and simple
Z K Y
The post below has been deleted. Click to close.
This post has been deleted. Click here to see post.
infiniteturtle
1131 posts
#15 • 2 Y
Y by Adventure10, Mango247
We can also use barycentrics on $\triangle ABC$. Let $Z=\omega _1\cap \omega _2$. Let $A=(1,0,0),B=(0,1,0),C=(0,0,1), D=(p:q:r),Z=(s:q:r)$ for some $p,q,r,s$.
Now if $\omega _1$ has the standard equation $a^2yz+b^2zx+c^2xy=(x+y+z)(ux+vy+wz)$, we get that $v=0$ and $u,w$ satisfy these two equations:
\[a^2qr+b^2rp+c^2qp=(p+q+r)(pu+rw)\text{ and }a^2qr+b^2rs+c^2qs=(s+q+r)(su+rw).\]To find $E$ we need to solve for $w$. One way to do so is to subtract the two equations, factor out $(p-s)$, then plug this equation into one of the originals; computation is not bad and is omitted. We get $w=\tfrac{a^2qr(p+q+r+s)+b^2prs+c^2pqs}{(p+q+r)(s+q+r)r}.$ Now if $E=(0,e,1-e)$ we get $a^2e(1-e)=w(1-e)\implies E=(0:w:a^2-w)$. Let's not plug $w$ into this; as we shall see, we're almost done. Now we need to find $Y$. Let $Y=(y,0,1-y)$, then \[\begin{vmatrix} y & 0 & 1-y \\ p & q & r \\ 0 & w & a^2-w \end{vmatrix}=0\iff w(p-y(p+q+r))+a^2qy=0.\]After multiplying by $r$, this is symmetric in $q$ and $r$, so we are done.
Z K Y
The post below has been deleted. Click to close.
This post has been deleted. Click here to see post.
math_pi_rate
1218 posts
#16 • 1 Y
Y by Adventure10
Nice problem. Here's my solution: Note that the problem states that $A$ lies on the radical axis of $\omega_1$ and $\omega_2$. This motivates us to consider the intersection of $\omega_1$ with $\overline{AB}$ and $\overline{AC}$. Call these intersections as $K$ and $L$, and let $\odot (DXY)=\gamma$. By the converse of Reim's Theorem on $\omega_1$ and $\gamma$, it suffices to show that $K$ lies on $\gamma$. Now, $$\angle KXD=\angle CFD-\angle FBX=\angle ALD-\angle ALS=\angle KLD$$This means that $X$ lies on $\odot (DKL)$. Similarly we can show that $Y$ lies on $\odot (DKL)$. Together these two facts give that $K$ lies on $\gamma$. Hence, done. $\blacksquare$
Z K Y
The post below has been deleted. Click to close.
This post has been deleted. Click here to see post.
Allen4567
55 posts
#17 • 1 Y
Y by Adventure10
Here is another solution using barycentric coordinates. We take $\triangle ABC$ as the reference triangle (Unlike the solution by @v_Enhance , which takes $\triangle DEF$).
Let $A=(1,0,0), B=(0,1,0), C=(0,0,1)$.
Let $\omega_1: -a^2yz-b^2zx-c^2xy + \left( x+y+z \right)\left( s_1 x +t_1 z \right) = 0$ and $\omega_2: -a^2yz-b^2zx-c^2xy + \left( x+y+z \right)\left( s_2 x +t_2 y \right) = 0$.
As their radical axis passes through $A$, $s_1=s_2=s$ and thus the equation of radical axis is $t_1z=t_2y$. Thus, let $D=(x_0:t_1:t_2)$.
By putting $x=0$ in $\omega_1$ , we obtain $E=(0:t_1:a^2-t_1)$.
Let $Y=(x_1,0,1-x_1)$.
$$\begin{vmatrix}
x_1 & 0 & 1-x_1\\ 
0 & t_{1} & a^2-t_1 \\
x_0 & t_{1} & t_{2}\\
\end{vmatrix}=0\implies x_1=\frac{x_0}{x_0+t_{1}+t_{2}-a^2}$$Similarly, if $X=(x_2,1-x_2,0),\ x_2=\frac{x_0}{x_0+t_{1}+t_{2}-a^2}$. Therefore, $x_1=x_2$, whence, $XY\parallel BC$.
$\blacksquare$
Z K Y
The post below has been deleted. Click to close.
This post has been deleted. Click here to see post.
yunseo
163 posts
#18 • 2 Y
Y by Adventure10, Mango247
Just angle chasing?! I feel like this might be too simple... Please let me know if I messed up.

Let $w_{1}$ intersect $BC$ at $Z (\not\equiv B)$, and $w_{2}$ intersect $BC$ at $W$. $BZWC$ cyclic by Pop. So, $\angle ACB = \angle AZW = \angle XDW$. So, $XZDW$ cyclic. Similarly, $YXDW$ cyclic, which means that $\frac{AY}{AX} = \frac{AC}{AB}$, completing our proof.
Z K Y
The post below has been deleted. Click to close.
This post has been deleted. Click here to see post.
Wizard_32
1566 posts
#19 • 2 Y
Y by Adventure10, Mango247
Moving points work too by showing the stronger result:
Stronger USA TST 2004/4 wrote:
Let $ABC$ be a triangle and $Z \in \overline{BC}$ be a point. Let $E \in \overline{BZ},F \in \overline{ZC}$ be two points such that $ZE \cdot ZB=ZF \cdot ZC.$ Let $D$ be an arbitrary point in $\overline{AZ}.$ Let $X=AB \cap FD$ and $Y=AC \cap ED.$ Then $XY \parallel BC.$
Move $D$ on $AZ.$ We need to show $X \mapsto D \mapsto Y$ is the same map as $X \mapsto X\infty_{BC} \cap AC,$ so need to check it for three positions. The case $D=A$ is obvious. The cases when $D=\infty_{AZ}$ and $D=AZ \cap E\infty_{AC}$ are easy by using the hypothesis $ZE \cdot ZB=ZF \cdot ZC.$
Z K Y
The post below has been deleted. Click to close.
This post has been deleted. Click here to see post.
AlastorMoody
2125 posts
#20 • 2 Y
Y by gamerrk1004, DPS
Solution
Z K Y
The post below has been deleted. Click to close.
This post has been deleted. Click here to see post.
Lcz
390 posts
#21 • 2 Y
Y by 606234, shafikbara48593762
Seems like a new (possibly cleaner? :O) bary sol. Let $D=(1,0,0), B=(0,1,0), C=(0,0,1), BC=a, DC=b, DB=c$. In the circle equation, let $\omega_1: (0,0,w)$, $\omega_2: (0,v,0)$, so that the radical axis of these two circles is $(t:w:v)$ and so we can set $A=(x:w:v)$. Plugging $x=0$ into $\omega_1$, we get $$a^2yz=(y+z)wz \implies (a^2-w)y=zw \implies E=(0:w:a^2-w)$$Intersecting the cevians $DE: (t:w:a^2-w)$ and $CA: (x:w:t)$ we get $Y=(x:w:a^2-w)$. Similarly, $X=(x:a^2-v:v)$ and we're clearly done.

They're all cevians :P
This post has been edited 1 time. Last edited by Lcz, Feb 15, 2021, 3:56 PM
Z K Y
The post below has been deleted. Click to close.
This post has been deleted. Click here to see post.
ppanther
160 posts
#22
Y by
[asy]
size(8cm);
defaultpen(fontsize(9pt));
import geometry;
pair A = dir(110), B = dir(180+30), C = dir(-30), D = (-0.2, 0.05), D1 = 1.3*D - 0.3*A;
path w1 = circumcircle(B, D, D1), w2 = circumcircle(C, D, D1);
pair E = intersectionpoints(line(B, C), w1)[1], F = intersectionpoints(line(B, C), w2)[0], G = intersectionpoints(line(A, B), w1)[0], H = intersectionpoints(line(A, C), w2)[0], X = extension(D, F, A, B), Y = extension(D, E, A, C);

draw(A--B--C--cycle, linewidth(0.9)); draw(w1^^w2, heavygray); draw(circumcircle(B, G, H), dotted); draw(circumcircle(G, H, D), dashed); draw(E--Y^^F--X, gray);

string[] names = {"$A$", "$B$", "$C$", "$D$", "$D_1$", "$E$", "$F$", "$G$", "$H$", "$X$", "$Y$"};
pair[] points = {A, B, C, D, D1, E, F, G, H, X, Y};
pair[] ll = {A, B, C, D, D1, E, F, G, H, X, Y};
for (int i=0; i<names.length; ++i)
	dot(names[i], points[i], dir(ll[i]));
[/asy]
Let $\omega_1, \omega_2$ intersect the sides $AB,AC$ again at $G,H$ as shown. Since
\[
AD \cdot AD_1 = AG \cdot AB = AH \cdot AC,
\]$BGHC$ is cyclic. The key observation is that $XYHDG$ is cyclic, which would clearly imply the problem (since then $\measuredangle AXY = \measuredangle GHY = \measuredangle ABC$). But this follows from
\[
\measuredangle XDH = \measuredangle FDH = \measuredangle BCH = \measuredangle BGH = \measuredangle XGH.
\]
This post has been edited 1 time. Last edited by ppanther, Feb 16, 2021, 3:36 AM
Reason: AD_1
Z K Y
The post below has been deleted. Click to close.
This post has been deleted. Click here to see post.
Mogmog8
1080 posts
#23 • 1 Y
Y by centslordm
Use barycentrics on triangle $DEF,$ and let $P=(p:m:n)$ and $A=(a_1:m:n).$ Notice that $$\omega_1:-a^2yz-b^2zx-c^2xy+(x+y+z)(\lambda z)$$where $\lambda=\frac{a^2mn+b^2pn+c^2pm}{(p+m+n)n}.$ Letting $B=(0,b_1,1-b_1),$ we find that $-a^2b_1(1-b_1)+\lambda(1-b_1)=0$ or $b_1=\frac{\lambda}{a^2}.$ Hence, $B=(0:\lambda:a^2-\lambda).$ Let $X=(x_1:0:x_2)$ and notice that \begin{align*}\begin{vmatrix}x_1&0&x_2\\a_1&m&n\\0&\lambda&a^2-\lambda\end{vmatrix}&=x_1\begin{vmatrix}m&n\\ \lambda&a^2-\lambda\end{vmatrix}+x_2\begin{vmatrix}a_1&m\\0&\lambda\end{vmatrix}\\&=-x_1(\lambda m+\lambda n-ma^2)+x_2a_1\lambda\\&=0\end{align*}so $X=(a_1:0:m+n-ma^2/\lambda).$ Similarly, $Y=(a_1:m+n-na^2/(\lambda n/m):0)$ so we are done. $\square$
This post has been edited 1 time. Last edited by Mogmog8, Dec 19, 2021, 4:30 PM
Z K Y
The post below has been deleted. Click to close.
This post has been deleted. Click here to see post.
Fakesolver19
106 posts
#24
Y by
Let $AB \cap \omega_1=X_1$ and $AC \cap \omega_2=Y_1$
Also let our second our intersection be $K$
Its trivial to see that $BCY_1X_1$ is cyclic by Pop
$$AK.AD=AX_1 \cdot AB=AY_1 \cdot AC$$Now angle chase finishes the problem
$$\angle X_1DY_1=\angle ABC=\angle YY_1X_1 \implies XX_1Y_1Y \text{is cyclic} \implies XY||BC$$
Z K Y
The post below has been deleted. Click to close.
This post has been deleted. Click here to see post.
REYNA_MAIN
41 posts
#25
Y by
This works right?
Shortage

Remark
Attachments:
Z K Y
The post below has been deleted. Click to close.
This post has been deleted. Click here to see post.
rafaello
1079 posts
#26
Y by
People like to overcomplicate this.
Quote:
Let $D$ be inside $\triangle ABC$ and $E$ on $AD$ different to $D$. Let $\omega_1$ and $\omega_2$ be the circumscribed circles of $\triangle BDE$ and $\triangle CDE$ respectively. $\omega_1$ and $\omega_2$ intersect $BC$ in the interior points $F$ and $G$ respectively. Let $X$ be the intersection between $DG$ and $AB$ and $Y$ the intersection between $DF$ and $AC$. Show that $XY$ is $\|$ to $BC$.
Let $P=\overline{DE}\cap\overline{BC}$. By PoP, $\frac{PB}{PG}=\frac{PC}{PF}$. Let $Y'$ be the intersection of line through $X$ parallel to $\overline{BC}$ and $\overline{DF}$. Then considering following composition of homotheties,
\begin{align*}
\overline{BC}\overset{P}{\rightarrow}\overline{GF}\overset{D}{\rightarrow}\overline{XY'}\overset{\overline{BX}\cap\overline{CY'}}{\rightarrow}\overline{BC},
\end{align*}thus $P,D,\overline{BX}\cap\overline{CY'}$ are collinear, hence $Y\equiv Y'$. We are done.
This post has been edited 1 time. Last edited by rafaello, Dec 22, 2021, 9:15 PM
Z K Y
The post below has been deleted. Click to close.
This post has been deleted. Click here to see post.
Mahdi_Mashayekhi
689 posts
#27
Y by
Let $\omega_1$ meet $AB$ at $P$ and $\omega_2$ meet $AC$ at $Q$ and Let our circles meet at $S$. Note that $AP.AB = AS.AD = AQ.AC$ so $PQCB$ is cyclic so by simple angle chasing we just need to prove $PQYX$ is cyclic. we have $\angle PDY = \angle ABC = \angle AQP \implies PQYD$ is cyclic. $\angle QDX = \angle ACB = \angle APQ \implies QPXD$ is cyclic so now we have $PQYDX$ is cyclic.
we're Done.
Z K Y
The post below has been deleted. Click to close.
This post has been deleted. Click here to see post.
ike.chen
1162 posts
#28
Y by
Let $\omega_1$ and $\omega_2$ meet again at $T$, $K = BT \cap \overline{DFX}$, and $L = CT \cap \overline{DEY}$.

Observe $$\measuredangle KDL = \measuredangle FDE = \measuredangle DFE + \measuredangle FED = \measuredangle DFC + \measuredangle BED$$$$= \measuredangle DTC + \measuredangle BTD = \measuredangle BTC = \measuredangle KTL$$so $DKTL$ is cyclic. This yields $$\measuredangle DFE = \measuredangle DFC = \measuredangle DTC = \measuredangle DTL = \measuredangle DKL$$which means $\overline{BEFC} \parallel KL$.

It's clear that $ABC$ and $DKL$ are centrally perspective at $T$. Now, Desargues Theorem implies $AB \cap DK = X$, $AC \cap DL = Y$, and $BC \cap KL = \infty_{BC}$ are collinear, which finishes. $\blacksquare$


Remark: This is the first time I've successfully applied Desargues :-D.
Z K Y
The post below has been deleted. Click to close.
This post has been deleted. Click here to see post.
crazyeyemoody907
450 posts
#29 • 1 Y
Y by Lcz
Let $K=\overline{AD}\cap\overline{BC}$. Then apply DDIT to $A,EFXY$, and project onto $\overline{BC}$, to obtain the involutive pairs $i:(E,X),(F,Y),(K,\overline{XY}\cap\overline{BC})$. Because $KE\cdot KX=KF\cdot KY$, $i$ must be an inversion at $K$ with that product as its power. Hence $K\overset{i}\leftrightarrow\infty_{BC}$, implying $\overline{XY}\cap\overline{BC}=\infty_{BC}$ as desired.
This post has been edited 4 times. Last edited by crazyeyemoody907, Sep 4, 2022, 12:38 AM
Z K Y
The post below has been deleted. Click to close.
This post has been deleted. Click here to see post.
john0512
4178 posts
#30
Y by
Let $AD$ intersect $BC$ at $P$. Note that the radical axis of $\omega_1$ and $\omega_2$ is line $AP$, so we have $$PB\cdot PE=PC\cdot PF.$$Save this equation for now.

We will employ barycentric coordinates. Suppose that $D=(p,q,r)$, $E=(0,e,1-e)$, and $F=(0,1-f,f).$ The equation for line $ED$ is $$[q(1-e)-er]x+[p(e-1)]y+epz=0,$$which we see $(p,q,r)$ and $(0,e,1-e)$ satsifies. Intersecting this with line $AC,$ we see that $Y$ satisfies $$[q(1-e)-er]x+epz=0.$$Note that $$\frac{AY}{CY}=\frac{\text{z coordinate of point Y}}{\text{x coordinate of point Y}}=\frac{er-q(1-e)}{ep}.$$By symmetry, we then have $$\frac{AX}{BX}=\frac{fq-r(1-f)}{fp}$$Therefore, $XY\parallel AB$ is equivalent to $$\frac{er-q(1-e)}{ep}=\frac{fq-r(1-f)}{fp}$$$$fer-fq+feq=feq-re+fer$$$$fq=re.$$
Now, we return to the equation $$PB\cdot PE=PC\cdot PF.$$Note that $P=(0:q:r)$, so $$PB=\frac{r}{q+r}a$$and $$PE=(\frac{q}{q+r}-e)a,$$so $$PB\cdot PE=a^2(\frac{r}{q+r})(\frac{q}{q+r}-e).$$Similarly, $$PC\cdot PF=a^2(\frac{q}{q+r})(\frac{r}{q+r}-f).$$Thus, we have $$(\frac{r}{q+r})(\frac{q}{q+r}-e)=(\frac{q}{q+r})(\frac{r}{q+r}-f)$$$$r(q-eq-er)=q(r-fq-fr)$$$$qr-eqr-er^2=qr-fqr-fq^2$$$$eqr+er^2=fqr+fq^2$$$$er(q+r)=fq(q+r)$$$$er=fq,$$as desired.
Z K Y
The post below has been deleted. Click to close.
This post has been deleted. Click here to see post.
KevinYang2.71
412 posts
#31 • 1 Y
Y by deduck
We use barycentric coordinates with $A=(1,0,0)$, $B=(0,1,0)$, and $C=(0,0,1)$. Let $D=:(p,q,r)$ and let the other intersection of $\omega_1$ and $\omega_2$ be $W=(t:q:r)$. Then $\omega_1$ is given by
\[
-a^2yz-b^2zx-c^2xy+(x+y+z)(u_1x+wz)=0.
\]It follows that
\begin{align*}
pu_1+rw&=a^2qr+b^2rp+c^2pq\\
tu_1+rw&=\frac{a^2pr+b^2rt+c^2tq}{t+q+r}.
\end{align*}In particular, note that if $\omega_2$ is given by
\[
-a^2yz-b^2zx-c^2xy+(x+y+z)(u_2x+vy)=0,
\]we have $u_1=u_2$ by symmetry (the only thing that is different is $rw$ being replaced with $qv$). Thus $rw=qv$. Letting $x=0$ in $\omega_1$ gives $y=\frac{w}{a^2}$ so $E=\left(0,\frac{w}{a^2},1-\frac{w}{a^2}\right)$. Let $X=:(x_1,0,x_2)$ so
\[
\begin{vmatrix}
x_1 & 0 & x_2\\
p & q & r\\
0 & \frac{w}{a^2} & 1-\frac{w}{a^2}
\end{vmatrix}
=0.
\]Solving for $x_2$, we get $x_2=1-\frac{pw}{qa^2}$. Letting $Y=:(y_1,y_2,0)$, we similarly get $y_2=1-\frac{pv}{ra^2}$. Since $rw=qv$, we have $x_2=y_2$ and $x_1=y_1$. Thus $\overrightarrow{XY}=(0,y_2,-x_2)$ is parallel to $\overrightarrow{BC}=(0,-1,1)$. $\square$
This post has been edited 1 time. Last edited by KevinYang2.71, Feb 20, 2024, 1:20 AM
Z K Y
The post below has been deleted. Click to close.
This post has been deleted. Click here to see post.
khanhnx
1618 posts
#32 • 1 Y
Y by GeoKing
Suppose that $\omega_1$ intersects $AB$ at $U,$ $\omega_2$ intersects $CA$ at $V,$ $\omega_1$ intersects $\omega_2$ at the second point $S$. We have $\overline{AU} \cdot \overline{AB} = \overline{AD} \cdot \overline{AS} = \overline{AV} \cdot \overline{AC}$. Then $B, C, U, V$ lie on a circle. We also have $\angle{UXD} = 180^{\circ} - \angle{ABC} - \angle{XFB} = \angle{UVC} - \angle{DVC} = \angle{UVD}$. Then $X, U, V, D$ lie on a circle. Similarly, $Y, U, V, D$ lie on a circle. So $X, Y, U, V$ lie on a circle. Hence $\angle{AYX} = \angle{AUV} = \angle{ACB}$ or $XY \parallel BC$
Z K Y
The post below has been deleted. Click to close.
This post has been deleted. Click here to see post.
cursed_tangent1434
573 posts
#33
Y by
Rather standard. We let $M$ and $N$ be the second intersections of circles $\omega_1$ and $\omega_2$ with $\overline{AB}$ and $\overline{AC}$ respectively. It is not hard to see that $BMCN$ is cyclic since,
\[AB \cdot AM = AD \cdot AP = AN \cdot AC\]Further, we can prove the following key claim.

Claim : Points $M$ , $N$ , $D$ , $X$ and $Y$ lie on a circle.
Proof : Simply note that,
\[\measuredangle MXD = \measuredangle BXF = \measuredangle BFX + \measuredangle XBF = \measuredangle EFD + \measuredangle MBF = \measuredangle CFD + \measuredangle MBC = \measuredangle CND + \measuredangle MNC = \measuredangle MND \]from which it is clear that $MXND$ is cyclic. A similar argument show that $MYND$ is also cyclic, proving the claim.

Now, the result follows by Reim's Theorem.
Z K Y
N Quick Reply
G
H
=
a